Re: [obm-l] Problema estranho

2017-07-08 Por tôpico Bruno Visnadi
Tecnicamente não dá para chamar de conjunto, quando há números repetidos. O correto seria Multiconjunto: https://pt.wikipedia.org/wiki/Multiconjunto Em 8 de julho de 2017 19:27, Luiz Antonio Rodrigues escreveu: > Olá, Otávio! > Desculpe a intromissão. Eu não sei como resolver seu problema, mas q

Re: [obm-l] Problema estranho

2017-07-08 Por tôpico Luiz Antonio Rodrigues
Olá, Otávio! Desculpe a intromissão. Eu não sei como resolver seu problema, mas quero aproveitá-lo para colocar uma questão que me atormenta desde a faculdade: pode existir um conjunto {1,1,1,2,3}? O número 1 não é único? Um abraço! Luiz On Jul 8, 2017 5:35 PM, "Otávio Araújo" wrote: Galera, que

Re: [obm-l] Problema da olimpiada hungara.

2017-05-23 Por tôpico Mauricio de Araujo
Obrigado!! -- Abraços, Mauricio de Araujo [oɾnɐɹɐ ǝp oıɔıɹnɐɯ] 2017-05-22 21:33 GMT-03:00 Pedro José : > Boa noite. > > Tentei da última vez escrever de uma forma simples, mas não deu, > tem muitas falhas, não vale, > > Na verdade, vai se formar um período a partir da a

Re: [obm-l] Problema da olimpiada hungara.

2017-05-22 Por tôpico Pedro José
Boa noite. Tentei da última vez escrever de uma forma simples, mas não deu, tem muitas falhas, não vale, Na verdade, vai se formar um período a partir da anomalia do algarismo das dezenas que é 1 e é a única vez que ele aparece. Depois será formado um período 023456789, que irá valer a princípio

Re: [obm-l] Problema da olimpiada hungara.

2017-05-19 Por tôpico Mauricio de Araujo
A resposta é: 0. -- Abraços, Mauricio de Araujo [oɾnɐɹɐ ǝp oıɔıɹnɐɯ] 2017-05-19 12:18 GMT-03:00 Jackson Sousa : > Onde conferimos a resposta da questão? > > > Em 17 de maio de 2017 09:16, Bernardo Freitas Paulo da Costa < > bernardo...@gmail.com> escreveu: > >> É bem ma

Re: [obm-l] Problema da olimpiada hungara.

2017-05-19 Por tôpico Jackson Sousa
Onde conferimos a resposta da questão? Em 17 de maio de 2017 09:16, Bernardo Freitas Paulo da Costa < bernardo...@gmail.com> escreveu: > É bem mais fácil. "Monte" o produto N*N como na escola. Vai ficar um > monte de "1" em cada linha e coluna. A 73ª coluna tem 73 "uns". > Agora, é só ver qua

Re: [obm-l] Problema da olimpiada hungara.

2017-05-17 Por tôpico Pedro José
Bom dia! Minha dúvida é de interpretação do português e não quanto a matemática. Quando se fala septuagésima terceira posição a partir do algarismo das unidades, fica dúvida inclusive ou exclusive? É mais fácil perguntar o algarismo de ordem 10^a. pois, dessa forma ficaria claro. Vou supor que é

Re: [obm-l] Problema da olimpiada hungara.

2017-05-17 Por tôpico Bernardo Freitas Paulo da Costa
É bem mais fácil. "Monte" o produto N*N como na escola. Vai ficar um monte de "1" em cada linha e coluna. A 73ª coluna tem 73 "uns". Agora, é só ver qual foi o "vai-um" da coluna anterior. E para isso tem que ver a anterior da anterior, mas (dica) não precisa ir muito longe. Abraços, -- Berna

Re: [obm-l] Problema da olimpiada hungara.

2017-05-16 Por tôpico Anderson Torres
N=99...9/9 = (10^2012-1)/9 9N = 10^2012-1 81N^2= 10^4024-2*10^2012+1 Agora tenta aplicar módulo 10^74: 81N^2= 10^4024-2*10^2012+1 81N^2=1 (mod 10^74) Agora teria que achar o "inverso" de 81 módulo 10^74, mas não parece fácil de cara. Outra forma seria usar alguma indução. Pelo que vi no P

[obm-l] Re: [obm-l] Problema Geometria

2017-05-09 Por tôpico Julio César Saldaña
Solução um pouco longa: - PB=PE - ABEC é inscritível => triângulo MEP = triângulo PEC (LAL). Por tanto No triângulo ABC, AB=AC. D é um ponto sobre o lado BC tal que BD=2CD. Se P é o ponto de AD tal que __ Si desea

Re: [obm-l] Problema

2017-04-15 Por tôpico Luiz Antonio Rodrigues
Olá, Bruno! Muito obrigado! Gostei muito da sua solução. Uma ótima Páscoa para você! Um abraço! Luiz On Apr 15, 2017 1:57 PM, "Bruno Visnadi" wrote: > Bom, o que importa não é quantas vezes elas comem por dia, e sim o quanto > elas comem durante cada dia. Digamos que todas as 16 vacas juntas com

Re: [obm-l] Problema

2017-04-15 Por tôpico Bruno Visnadi
Bom, o que importa não é quantas vezes elas comem por dia, e sim o quanto elas comem durante cada dia. Digamos que todas as 16 vacas juntas comam N quilos de ração por dia, e temos 62N quilos ao total. Após 14 dias, sobram 48N quilos. Então ele vende 4 vacas, e a taxa de consumo passa a ser 3N/4 qu

Re: [obm-l] Problema de geometria.

2016-11-02 Por tôpico Bruno Visnadi
De acordo com o site http://objetoseducacionais2.mec.gov.br/bitstream/handle/mec/10396/geo0500.htm Flecha é um segmento de reta que une o ponto médio de uma corda ao ponto médio do arco correspondente. Em 2 de novembro de 2016 20:06, Tarsis Esau escreveu: > Se essas "flechas" forem lados o triâ

Re: [obm-l] Problema de geometria.

2016-11-02 Por tôpico Tarsis Esau
Se essas "flechas" forem lados o triângulo não existe. Em 02/11/2016 6:16 PM, "Esdras Muniz" escreveu: > O que são essas "flechas"? > > Em 2 de novembro de 2016 17:57, Douglas Oliveira de Lima < > profdouglaso.del...@gmail.com> escreveu: > >> Olá amigos , preciso de uma ajuda na seguinte questão

Re: [obm-l] Problema de geometria.

2016-11-02 Por tôpico Esdras Muniz
O que são essas "flechas"? Em 2 de novembro de 2016 17:57, Douglas Oliveira de Lima < profdouglaso.del...@gmail.com> escreveu: > Olá amigos , preciso de uma ajuda na seguinte questão, na verdade a > resolução porque já tentei muita coisa, já aprendi muita coisa com ela, mas > mesmo assim não a re

Re: [obm-l] Problema

2016-10-08 Por tôpico Matheus Herculano
Para de me mandar isdo Em 8 de out de 2016 08:12, "regis barros" escreveu: > Bom dia > segue o problema > se x^y = 2 e y^x = 3, encontrar os valores de x e y. > > Grato > > Regis > > > Em Quarta-feira, 5 de Outubro de 2016 18:01, vinicius raimundo < > vini.raimu...@gmail.com> escreveu: > > > Obr

Re: [obm-l] Problema

2016-10-08 Por tôpico Ralph Teixeira
Oi, Regis. Eu acho (acho!) que nao dah para resolver esse sistema no braco com as funcoes elementares usuais. Eliminando uma das variaveis, recai em algo do tipo: ln(lny)+(ln2)/y=ln(ln3) ou ln(lnx)+(ln3)/x=ln(ln2) E, ateh onde eu consigo pensar, equacoes desse tipo nao se resolvem no braco. Ago

Re: [obm-l] Problema

2016-07-11 Por tôpico Lucas Melo
Muito obrigado!! Enviado do meu iPhone > Em 8 de jul de 2016, às 13:47, Pedro José escreveu: > > Boa tarde! > > Primeiro, entendo que houve um erro no enunciado do problema, destacado em > amarelo. Deveria com raízes inteiras a1 e b1 e não a1 e a2 como escrito. > O problema é meio controve

Re: [obm-l] Problema

2016-07-08 Por tôpico Pedro José
Boa tarde! Primeiro, entendo que houve um erro no enunciado do problema, destacado em amarelo. Deveria com raízes inteiras a1 e b1 e não a1 e a2 como escrito. O problema é meio controverso Pois não existe apenas uma equação. Qual o critério para a1 e b1? Se for assim: E dada uma equacao do segund

Re: [obm-l] Problema sobre trilha com subidas e descidas

2016-05-30 Por tôpico Pedro José
Boa tarde! Não sou professor. Sou leigo. Sou só um adimirador da matemática. Saudações, PJMS Em 28 de maio de 2016 13:38, Marcelo Gomes escreveu: > Olá professor Pedro, muito obrigado! > > Pois é, na minha cabeça, 3Km/h de velocidade, indicavam um trecho de 3 Km, > percorridos no tempo de 1 ho

Re: [obm-l] Problema sobre trilha com subidas e descidas

2016-05-28 Por tôpico Marcelo Gomes
Olá professor Pedro, muito obrigado! Pois é, na minha cabeça, 3Km/h de velocidade, indicavam um trecho de 3 Km, percorridos no tempo de 1 hora. Outra coisa que não havia compreendido é a questão da volta. Na minha cabeça, a trilha teria início no ponto A e fim em um ponto B. Obrigado pelas explic

Re: [obm-l] Problema sobre trilha com subidas e descidas

2016-05-27 Por tôpico Pedro José
Boa tarde! Seja *a* o trecho de subida e *b* o trecho de descida na ida para cahoeira teremos que *b *será o trecho de subida e *a* o trecho de descida na volta. Portanto: a/3 + b/4 = 3 2/3 a/4 + b/3 = 3 1/3 Resolvendo o sistema a = 8 km e b = 4km. Portanto o comprimento de cada perna é 12 km.

RE: [obm-l] Problema 6 da OBM de 2002

2015-10-12 Por tôpico Esdras Muniz
Em qual EUREKA está a solução deste problema? -Mensagem Original- De: "Bernardo Freitas Paulo da Costa" Enviada em: ‎12/‎10/‎2015 12:29 Para: "Lista de E-mails da OBM" Assunto: Re: [obm-l] Problema 6 da OBM de 2002 2015-10-12 0:31 GMT-03:00 Gabriel Tostes : >

Re: [obm-l] Problema 6 da OBM de 2002

2015-10-12 Por tôpico Bernardo Freitas Paulo da Costa
2015-10-12 0:31 GMT-03:00 Gabriel Tostes : > Mostre que não podemos formar mais que 4096 sequências binárias de tamanho 24 > tal que quaisquer 2 diferem em ao menos 8 posições. > Não consegui entender a resolução na Eureka. Alguém pode resolvê-lo? Eu não sei se conheço alguma solução além da do F

[obm-l] Re: [obm-l] Re: [obm-l] Re: [obm-l] Re: [obm-l] Problema muito bacana de teoria dos números

2015-08-08 Por tôpico Douglas Oliveira de Lima
Bom , vamos lá: 1)Como N possui 12 divisores, temos que 1 será o menor e N será o maior. 2)Usando uma propriedade bem conhecida teremos dk.d(13-k)=t, ou seja o divisor de indice k e o de índice 13-k. 3)Como o divisor de índice d4-1 é igual a (d1+d2+d4)d8, teremos que d1+d2+d4 é divisor também lo

[obm-l] Re: [obm-l] Re: [obm-l] Re: [obm-l] Problema muito bacana de teoria dos números

2015-08-07 Por tôpico Pedro José
Boa tarde! Saulo, Se 2 e 3 são divisores 6 também será. Achei esse problema casca grossa. Saudações, PJMS Em 6 de agosto de 2015 23:25, Mauricio de Araujo < mauricio.de.ara...@gmail.com> escreveu: > N = 1989. > > Em 6 de agosto de 2015 14:50, saulo nilson > escreveu: > >> d4-1=11 >> d4=12 >>

[obm-l] Re: [obm-l] Re: [obm-l] Problema muito bacana de teoria dos números

2015-08-06 Por tôpico Mauricio de Araujo
N = 1989. Em 6 de agosto de 2015 14:50, saulo nilson escreveu: > d4-1=11 > d4=12 > d1=1 > d2=2 > d3= > d11=(1+2+12)d8=15*17=255 > 1,2,3,12,13,14,15,17,18,19,255, produto deles. > > 2015-08-06 13:14 GMT-03:00 Mauricio de Araujo < > mauricio.de.ara...@gmail.com>: > >> Um número natural N tem exata

[obm-l] Re: [obm-l] Problema muito bacana de teoria dos números

2015-08-06 Por tôpico saulo nilson
d4-1=11 d4=12 d1=1 d2=2 d3= d11=(1+2+12)d8=15*17=255 1,2,3,12,13,14,15,17,18,19,255, produto deles. 2015-08-06 13:14 GMT-03:00 Mauricio de Araujo : > Um número natural N tem exatamente 12 divisores (incluindo 1 e N), tais > que, colocados em ordem crescente temos d1 < d2 < d3 < ... < d12. > Sabe

[obm-l] Re: [obm-l] Problema muito bacana de teoria dos números

2015-08-06 Por tôpico saulo nilson
d4-1=11 d4=12 d1=1 d2=2 d3= d11=(1+2+12)d8=15*17=255 1,2,3,12,13,14,15,17,18,19,255,256 2015-08-06 13:14 GMT-03:00 Mauricio de Araujo : > Um número natural N tem exatamente 12 divisores (incluindo 1 e N), tais > que, colocados em ordem crescente temos d1 < d2 < d3 < ... < d12. > Sabe-se que o di

Re: [obm-l] Problema

2015-07-14 Por tôpico Benedito Tadeu V. Freire
Rogério, Olá. Muito obrigado. Benedito -- Open WebMail Project (http://openwebmail.org) -- Original Message --- From: Rogerio Ponce To: "obm-l@mat.puc-rio.br" Sent: Tue, 7 Jul 2015 19:43:31 -0300 Subject: Re: [obm-l] Problema > Ola' Benedito, > Em mo

RE: [obm-l] Problema

2015-07-09 Por tôpico benedito freire
Obrigado Gugu -Mensagem Original- De: "g...@impa.br" Enviada em: ‎09/‎07/‎2015 17:08 Para: "obm-l@mat.puc-rio.br" Cc: "fe...@impa.br" Assunto: Re: [obm-l] Problema Caro Benedito, Encaminho abaixo a solução do Renan Finder, que é ex-olímpico e

RE: [obm-l] Problema

2015-07-09 Por tôpico benedito freire
Obrigado Gugu -Mensagem Original- De: "g...@impa.br" Enviada em: ‎09/‎07/‎2015 17:08 Para: "obm-l@mat.puc-rio.br" Cc: "fe...@impa.br" Assunto: Re: [obm-l] Problema Caro Benedito, Encaminho abaixo a solução do Renan Finder, que é ex-olímpico e

Re: [obm-l] Problema

2015-07-09 Por tôpico gugu
Caro Benedito, Encaminho abaixo a solução do Renan Finder, que é ex-olímpico e aluno do IMPA, e mostra que A tem estratégia para ganhar: Chamamos de "classe n" o conjunto dos números remanescentes que são congruentes a n módulo 5. O jogador A vence se tentar minimizar a quantidade |#

Re: [obm-l] Problema

2015-07-07 Por tôpico Rogerio Ponce
rtence ao par estranho, e entao apaga o "0" associado. Ou seja, sempre que "B" apagar um numero, "A" apaga o complemento. Ao final, sempre sobrara' um par complementar. []'s Rogerio Ponce 2015-07-06 14:39 GMT-03:00 benedito freire : > Qual é realmente a estra

RE: [obm-l] Problema

2015-07-06 Por tôpico benedito freire
Qual é realmente a estratégia para vencer? -Mensagem Original- De: "Mauricio de Araujo" Enviada em: ‎01/‎07/‎2015 14:24 Para: "obm-l@mat.puc-rio.br" Assunto: Re: [obm-l] Problema ​ou melhor, A deve evitar enquanto puder apagar algum múltiplo de 5.​ Em 1 de

Re: [obm-l] Problema

2015-07-01 Por tôpico Mauricio de Araujo
de cabarem >>> todos os números. Necessita de reanálise. >>> ---------- Mensagem encaminhada -- >>> De: Pedro José >>> Data: 1 de julho de 2015 10:54 >>> Assunto: Re: [obm-l] Problema >>> Para: obm-l@mat.puc-rio.br >>> >>> >

Re: [obm-l] Problema

2015-07-01 Por tôpico Mauricio de Araujo
stá errado o jogador pode escolher a sobra de E ou F antes de cabarem >> todos os números. Necessita de reanálise. >> -- Mensagem encaminhada -- >> De: Pedro José >> Data: 1 de julho de 2015 10:54 >> Assunto: Re: [obm-l] Problema >> Pa

Re: [obm-l] Problema

2015-07-01 Por tôpico Mauricio de Araujo
é > Data: 1 de julho de 2015 10:54 > Assunto: Re: [obm-l] Problema > Para: obm-l@mat.puc-rio.br > > > > Bom dia! > > > E={1,6,11,16,21,26} e F= {4,9,14,19,24} Para qualquer par (a,b) com a Ɛ E > e b Ɛ F ==> a + b ≡ 0 (mod5). > G= {2, 7, 12, 17, 22,27} e H = {3, 8

Re: [obm-l] Problema

2015-07-01 Por tôpico Pedro José
Bom dia! E={1,6,11,16,21,26} e F= {4,9,14,19,24} Para qualquer par (a,b) com a Ɛ E e b Ɛ F ==> a + b ≡ 0 (mod5). G= {2, 7, 12, 17, 22,27} e H = {3, 8, 13, 18, 23} Para qualquer (a,b) com a Ɛ G e b Ɛ H ==> a + b ≡ 0 (mod5). J= {5, 15, 20, 25} Para qualquer par (a,b) com a,b Ɛ J==> a + b ≡ 0 (mod5

Re: [obm-l] Problema

2015-06-26 Por tôpico Mauricio de Araujo
Existe uma solução para este problema na revista Eureka no. 5. Em 22 de junho de 2015 18:32, Douglas Oliveira de Lima < profdouglaso.del...@gmail.com> escreveu: > Olá caros colegas, gostaria de uma ajuda no seguinte problema: > > Em uma reta há 1999 bolinhas. Algumas são verdes e as demais > azui

[obm-l] Re: [obm-l] Problema da 18ª Olimpíada de Maio

2015-06-26 Por tôpico Rogerio Ponce
Ola' Mariana, trace por M uma perpendicular ao lado BC, e chame de "E" sua intersecao com DB. Chame de "F" a intersecao de DM com CE. Por construcao, o triangulo EBC e' isosceles. Como CD e' perpendicular 'a CA, entao CD e' bissetriz ( externa ) do angulo entre o lado CD e o prolongamento do lado

Re: [obm-l] Problema de Desigualdade

2015-06-11 Por tôpico Alexandre Antunes
Boa tarde, Pensei em fazer essa prova por indução ... Ainda não consegui parar para finalizar. Achei que era um caminho possível!!! Em 11/06/2015 14:28, "Douglas Oliveira de Lima" < profdouglaso.del...@gmail.com> escreveu: > Então não é trabalhoso, mas (a/b)^2 >= 1 + a/b - b/a não deveria ser >

Re: [obm-l] Problema de Desigualdade

2015-06-11 Por tôpico Douglas Oliveira de Lima
Então não é trabalhoso, mas (a/b)^2 >= 1 + a/b - b/a não deveria ser provado? Desenvolvendo da pra ver que é, neste caso tem mais conta pra fazer. Forte abraço Douglas Oliveira. Em 10 de junho de 2015 12:00, Alexandre Antunes < prof.alexandreantu...@gmail.com> escreveu: > > Bom dia, > > Estou

Re: [obm-l] Problema de Desigualdade

2015-06-10 Por tôpico Alexandre Antunes
Bom dia, Estou no trabalho, mas vou arriscar a minha primeira resposta no grupo. Desenvolvi os dois lados da expressao. (a/b)^2 + (b/c)^2 + (c/a)^2 = 3 + (a/b + b/c + c/a) - (b/a + c/b + a/c) Como (a/b)^2 >= 1 + a/b - b/a O mesmo para os demais termos Fica provado a proposição. O que acham de

Re: [obm-l] Problema de Desigualdade

2015-06-10 Por tôpico Pacini Bores
Ok Mariana. Abraços Pacini Em 9 de junho de 2015 21:11, Mariana Groff escreveu: > Oi Pacini, > Fiz do seguinte modo: > f (x)=x^2-x+1/x>=1 <=> x^3-x^2+1>=x <=> x^3-x^2-x+1>=0 <=>x^2 > (x-1)-(x-1)>=0 <=> (x^2-1)(x-1)>=0 > O que podemos ver que é verdade, analisando ambos os casos: em que x>=1 e

Re: [obm-l] Problema de Desigualdade

2015-06-09 Por tôpico Mariana Groff
Oi Pacini, Fiz do seguinte modo: f (x)=x^2-x+1/x>=1 <=> x^3-x^2+1>=x <=> x^3-x^2-x+1>=0 <=>x^2 (x-1)-(x-1)>=0 <=> (x^2-1)(x-1)>=0 O que podemos ver que é verdade, analisando ambos os casos: em que x>=1 e o caso em que 0 escreveu: > Oi Mariana, > > Determinei o mínimo da função usando a derivada.

Re: [obm-l] Problema de Desigualdade

2015-06-09 Por tôpico Pacini Bores
Oi Mariana, Determinei o mínimo da função usando a derivada. Não entendi o seu caminho, pois a função é f(x) = x^2-x+1/x. Abraços Pacini Em 9 de junho de 2015 18:09, Mariana Groff escreveu: >Oi Pacini, > Compreendi seu raciocínio. Para provar que f(x)>=1, basta analisarmos que > (x^2-1)(

Re: [obm-l] Problema de Desigualdade

2015-06-09 Por tôpico Mariana Groff
Oi Pacini, Compreendi seu raciocínio. Para provar que f(x)>=1, basta analisarmos que (x^2-1)(x-1)>=0, o que verifica-se pois se x>=1, o produto é claramente não-negativo e se 0 escreveu: > Oi Mariana, > Observe que provar a desigualdade pedida é equivalente provar que : > > {(a/b)^2-a/b+b/a

Re: [obm-l] Problema de Desigualdade

2015-06-09 Por tôpico Pacini Bores
Oi Mariana, Observe que provar a desigualdade pedida é equivalente provar que : {(a/b)^2-a/b+b/a} + {(b/c)^2-b/c+c/b} +{(c/a)^2-c/a+a/c} >=3, ok ? Agora façamos o seguinte : Seja f(x)= x^2-x+1/x, verifique que para x>0 o valor mínimo de f é 1. Donde teremos a desigualdade provada. Estou c

Re: [obm-l] Problema de Desigualdade

2015-06-08 Por tôpico Raphael Aureliano
MA>=MG LE=(a/b+b/c+c/a)^2>=(3cbrt(abc/abc))^2 =9 Por Cauchy LD=(a+b+c)(1/a+1/b+1/c)<=(sqrt(a/a) +sqrt(b/b)+sqrt(c/c))^2 =9 LE>=9>=LD Em 08/06/2015 19:20, "Mariana Groff" escreveu: > Boa Noite, > > (British Mathematical Olympiad - Round 2 - 2005) > Sejam a,b e c reais positivos. > Prove que > >

Re: [obm-l] Problema de Desigualdade

2015-06-08 Por tôpico Raphael Aureliano
Ah não, desculpa, errei em Cauchy ... Att. Raphael Em 08/06/2015 20:27, "Raphael Aureliano" escreveu: > MA>=MG > LE=(a/b+b/c+c/a)^2>=(3cbrt(abc/abc))^2 =9 > > Por Cauchy > LD=(a+b+c)(1/a+1/b+1/c)<=(sqrt(a/a) +sqrt(b/b)+sqrt(c/c))^2 =9 > > LE>=9>=LD > Em 08/06/2015 19:20, "Mariana Groff" > escr

Re: [obm-l] Problema de Geometria

2015-04-30 Por tôpico Marcos Martinelli
Não é uma solução com geometria pura. ___ Lema) Um quadrilátero XYZW é inscritível se somente se XZ * YW = XY * ZW + XW * YZ . Solução) Sejam p, r, a, b e c a notação usual de um triângulo ABC qualquer. Seja F o ponto de tangência

Re: [obm-l] Problema de Geometria

2015-04-30 Por tôpico Douglas Oliveira de Lima
Então Mariana, questão muito boa, eu fiz aqui por números complexos, meio mecânico, não vi ainda uma solução por plana somente. Vou tentar mais um pouco. Abraços Douglas Oliveira. Em 26 de abril de 2015 16:25, Mariana Groff escreveu: > Boa tarde, > Alguém poderia me ajudar no problema a seguir?

Re: [obm-l] Problema das caixas

2015-04-23 Por tôpico Mariana Groff
Entendi. Obrigada Em 22/04/2015 10:50, "Esdras Muniz" escreveu: > Acho q é 1169. > > Em 21 de abril de 2015 15:21, Mariana Groff < > bigolingroff.mari...@gmail.com> escreveu: > >> Boa tarde, >> Alguém poderia me ajudar no problema a seguir? >> >> Temos 27 caixas em fila; cada uma delas contém pel

Re: [obm-l] Problema das caixas

2015-04-22 Por tôpico Esdras Muniz
Acho q é 1169. Em 21 de abril de 2015 15:21, Mariana Groff escreveu: > Boa tarde, > Alguém poderia me ajudar no problema a seguir? > > Temos 27 caixas em fila; cada uma delas contém pelo menos 12 bolinhas. A > operação permitida é transferir uma bolinha de uma caixa para sua vizinha > da direita

Re: [obm-l] Problema das caixas

2015-04-22 Por tôpico Rogerio Ponce
Ola' Mariana, como as bolinhas "andam" somente para a direita, a ultima caixa 'a direita e' o destino de todas as bolinhas. Repare que temos que esvaziar primeiramente a caixa mais 'a esquerda, em seguida a proxima do seu lado direito, e assim sucessivamente, caso contrario o fluxo seria interrompi

Re: [obm-l] Problema IMO

2015-03-11 Por tôpico Israel Meireles Chrisostomo
Obrigado mesmo, vlw Carlos |Victor Em 10 de março de 2015 22:14, Carlos Victor escreveu: > Oi Israel, no link > > > http://www.artofproblemsolving.com/wiki/index.php/1985_IMO_Problems/Problem_4, > vc encontra a solução, ok ? > > > Abraços > > > Carlos Victor > > > > Em 10 de março de 2015 21:

Re: [obm-l] Problema IMO

2015-03-10 Por tôpico Carlos Victor
Oi Israel, no link http://www.artofproblemsolving.com/wiki/index.php/1985_IMO_Problems/Problem_4, vc encontra a solução, ok ? Abraços Carlos Victor Em 10 de março de 2015 21:46, Israel Meireles Chrisostomo < israelmchrisost...@gmail.com> escreveu: > Alguém poderia me ajudar nessa questão

Re: [obm-l] Problema interessante de EDO

2014-12-21 Por tôpico saulo nilson
y(x)=A(x)senx+B(x)cosx y(x)=0 sen(x+u)=0 x+u=2npi x=2npi-u que sao infinitos valores de n para obter x. 2014-12-19 19:50 GMT-02:00 Artur Costa Steiner : > Seja g uma função contínua em [a, oo) tal que, para todo x neste > intervalo, tenhamos g(x) > m > 0. Mostre que, se y é solução da EDO > > y''

Re: [obm-l] Problema interessante de EDO

2014-12-21 Por tôpico saulo nilson
y=A(x)senx y´=A´senx+Acosx y"=A"cosx+A´cosx+A´cosx-Asenx A"+2A´=0 A´=u u´+2u=0 lnu=-2x+c u=Ce^(-2x) A(x)=C1e^(-2x)+C2 y(x)=(C1e^(-2x)+C2)senx=0 x=2npi que corresponde a infinitos zeros 2014-12-19 19:50 GMT-02:00 Artur Costa Steiner : > Seja g uma função contínua em [a, oo) tal que, para todo x nes

[obm-l] Re: [obm-l] Re: [obm-l] Problema de análise

2014-11-14 Por tôpico saulo nilson
-1: > Hmmm... Deu vontade de olhar para g(x)=n.ln[f(x)] + m ln[f(-x)], cuja > derivada é g'(x)=n.f'(x)/f(x) - m. f´(-x)/f(-x). Ou seja, a condição pedida > passaria a ser g´(c)=0. > > Como g(0)=0 independentemente de m e n, basta achar um outro ponto d onde > g(d)=0 para usar um Rolle. Ou seja, vo

[obm-l] Re: [obm-l] Problema de análise

2014-11-12 Por tôpico Ralph Teixeira
Hmmm... Deu vontade de olhar para g(x)=n.ln[f(x)] + m ln[f(-x)], cuja derivada é g'(x)=n.f'(x)/f(x) - m. f´(-x)/f(-x). Ou seja, a condição pedida passaria a ser g´(c)=0. Como g(0)=0 independentemente de m e n, basta achar um outro ponto d onde g(d)=0 para usar um Rolle. Ou seja, você quer mostrar

Re: [obm-l] Re: [obm-l] Problema de análise

2014-11-12 Por tôpico Artur Costa Steiner
Oh, de fato está errado. f é diferenciável em (-1, 1) Obrigado. Artur Costa Steiner > Em 12/11/2014, às 07:18, Rafael Dumas escreveu: > > O enunciado está correto? c e -c são simétricos, um é positivo e outro > negativo ou c = 0. Mas o enunciado afirma que f só é diferenciável em (0,

[obm-l] Re: [obm-l] Problema de análise

2014-11-12 Por tôpico Rafael Dumas
O enunciado está correto? c e -c são simétricos, um é positivo e outro negativo ou c = 0. Mas o enunciado afirma que f só é diferenciável em (0, 1). Em 12 de novembro de 2014 00:07, Artur Costa Steiner escreveu: > Oi amigos, > > Ainda não consegui resolver este não. Alguém pode colaborar? > > Su

Re: [obm-l] problema real - off topic

2014-11-08 Por tôpico Ralph Teixeira
Ah, olha soh: as combinacoes lineares de 38 e 56 (com coeficientes inteiros) abaixo de 300 sao: Sem usar 56: 38, 76, 114,152,190, 228, 266; Com um 56: 56, 94, 132, 170, 208, 246, 284; Com dois 56: 112,150,188, 226, 264; Com tres 56: 168, 206, 244, 282; Com quatro 56: 224, 262, 300; Com cinco 56: 28

Re: [obm-l] problema real - off topic

2014-11-08 Por tôpico Ralph Teixeira
Hmmm... Acho que eh um tiquinho mais complicado, se a gente levar em conta misturas de orientacoes. Por exemplo: voce poderia pegar a chapa 300x120 e dividi-la em 224x120 + 76x120. A primeira vira (4x56)x(3x38) = 12 chapas, e a segunda vira (2x38)x(2x56)=4 chapas. Entao em teoria eh possivel fazer

Re: [obm-l] problema real - off topic

2014-11-08 Por tôpico saulo nilson
Preciso cortar chapas de 38cm x 56cm e gostaria de saber qual dos tamanhos de chapa abaixo seria o melhor (ou seja, menor perda) 200cm x 100cm 200cm x 120cm 300cm x 100cm 300cm x 120cm 168*76 sobra 24*168+100*32=7232cm^2 168 *114 sobra 6*168+120*32=4848 280*76 sobra 20*100+24*280=8720 280*114 so

Re: [obm-l] problema real - off topic

2014-11-08 Por tôpico saulo nilson
Preciso cortar chapas de 38cm x 56cm e gostaria de saber qual dos tamanhos de chapa abaixo seria o melhor (ou seja, menor perda) 200cm x 100cm 200cm x 120cm 300cm x 100cm 300cm x 120cm 300*120 com e melhor 2014-11-08 12:06 GMT-02:00 Rogerio Ponce : > Ola' Hermann, > escolha uma das chapas de 12

Re: [obm-l] problema real - off topic

2014-11-08 Por tôpico Rogerio Ponce
Ola' Hermann, escolha uma das chapas de 120cm de largura. Se for a de 200cm de comprimento, a divisao do comprimento por 5 (e da largura por 2) gera retangulos de 40cmx60cm. Portanto voce obtera' 10 pedacos do tamanho desejado. Se for a de 300cm, a divisao do comprimento por 5 (e da largura por 3

Re: [obm-l] Problema de pilhas

2014-11-07 Por tôpico Ralph Teixeira
Uma maneira que satisfaz as condições do enunciado é com 30 pilhas: 1,3,5,7,9,...,59 Ao dividir qualquer pilha em duas, tem que aparecer um ímpar menor, então haverá repetição. Agora temos que mostrar que não há maneira mais eficiente que esta... Suponha, por contradição, que você conseguiu uma

Re: [obm-l] Problema de pilhas

2014-11-06 Por tôpico saulo nilson
k=1 450,450 2014-11-02 14:08 GMT-02:00 Mariana Groff : > Boa Tarde, > Alguém poderia, por favor, me auxiliar neste problema? > > Devemos distribuir 900 pedras em k pilhas, de modo que sejam satisfeitas > as condições a seguir: > (i) todas as pilhas têm quantidades distintas de pedras; > (ii) se d

Re: [obm-l] Problema de pilhas

2014-11-06 Por tôpico saulo nilson
k(1+(k-1)r+1)/2=900 rk^2+k(2-r)-1800=0 delta=(2-r)^2+r7200 r=2 o menor r k=30 2014-11-02 14:08 GMT-02:00 Mariana Groff : > Boa Tarde, > Alguém poderia, por favor, me auxiliar neste problema? > > Devemos distribuir 900 pedras em k pilhas, de modo que sejam satisfeitas > as condições a seguir: > (i

[obm-l] Re: [obm-l] problema de análise combinatória.

2014-11-06 Por tôpico Pedro José
Boa tarde! Use o princípio da multiplicação. Para goleiro, quantas opções temos? x Para lateral direito quantas opções? y Para zagueiro direito? E assim por diante até chegar ao ponta esquerda. Multiplique tudo. Sds, PJMS Em 6 de novembro de 2014 14:55, Mauricio Barbosa escreveu: > Boa tar

Re: [obm-l] Problema de Geometria

2014-11-03 Por tôpico Douglas Oliveira de Lima
Olá Mariana, eu vi uma solução fazendo alguns traçados algum dia na minha vida, se me lembro bem foi em um dos livros do Ross Honsberger, ela é difícil, mas vou tentar escreve-la. Faça uma figura e acompanhe ok?? 1)Desenhe o triângulo ABC e tome um ponto P externamente tal que PA=PD, de forma que

Re: [obm-l] Problema de Geometria

2014-11-03 Por tôpico Carlos Victor
Oi Mariana, Seja x o ângulo DCA . Aplicando a lei dos senos nos triângulos ACD e BCD , vc encontrará a seguinte relação : senx = 2sen(x+20).cos80. Transformando em soma teremos : senx = sen(x+100) + sen(x-60). Jogando para a esquerda o sen(x-60), teremos senx - sen(x-60) = sen(x+100); ou s

[obm-l] Re: [obm-l] Re: [obm-l] Re: [obm-l] Re: [obm-l] Re: [obm-l] Re: [obm-l] Problema sobre existência de subconjunto divisível

2014-11-02 Por tôpico Pedro Nascimento
Achei uma solucao aqui : http://mathoverflow.net/questions/16721/egz-theorem-erdos-ginzburg-ziv Em 22 de outubro de 2012 09:02, terence thirteen escreveu: > Lembrei vagamente deste problema, mas acho que ele é mais complicado > do que imaginamos. > > Lembro que num livro de Ross Honsberger, talv

[obm-l] Re: [obm-l] Problema de Álgebra

2014-10-29 Por tôpico saulo nilson
a) 729 b) 9216=(96)^2 94^2=8836 tem mais de uma manneira se n>12 2014-10-29 18:56 GMT-02:00 Mariana Groff : > Boa tarde, > Não consigo resolver o problema a seguir, alguém poderia me ajudar? > > O inteiro n é o produto de dois inteiros positivos. Prove que > > (a) é possível escrever dois algari

[obm-l] Re: [obm-l] Problema de Álgebra

2014-10-29 Por tôpico Mariana Groff
Perdão, Invés de n ser o produto de dois inteiros positivos, n é o produto de dois inteiros positivos consecutivos. Em 29 de outubro de 2014 20:03, escreveu: >Cara Mariana, >Acho que há algum problema com o enunciado. Seja n=122=2.61. Se > escrevemos dois algarismos após o algarismo das

Re: [obm-l] Problema de Álgebra

2014-10-29 Por tôpico gugu
Cara Mariana, Acho que há algum problema com o enunciado. Seja n=122=2.61. Se escrevemos dois algarismos após o algarismo das unidades de n obtemos um número entre 12200 e 12299. Como 110^2=12100<12200 e 111^2=12321>12299, nenhum desses números é um quadrado perfeito. Abraços,

[obm-l] Re: [obm-l] Re: [obm-l] Problema de Álgebra

2014-10-20 Por tôpico Mariana Groff
Entendi, Muito obrigada! Em 20 de outubro de 2014 18:12, Pacini Bores escreveu: > Oi Mariana, > > Observe que c =-(a+b) e levando na expressão original teremos : > > a^4+b^4 + c^4 = a^4+b^4+(a+b)^4. Desenvolvendo esta expressão , teremos > como resultado : > > 2(a^4+b^4+2a^3b+3a^2b^2+2ab^3) = 2(

[obm-l] Re: [obm-l] Problema de Álgebra

2014-10-20 Por tôpico Pacini Bores
Oi Mariana, Observe que c =-(a+b) e levando na expressão original teremos : a^4+b^4 + c^4 = a^4+b^4+(a+b)^4. Desenvolvendo esta expressão , teremos como resultado : 2(a^4+b^4+2a^3b+3a^2b^2+2ab^3) = 2(a^2+b^2+ab)^2, ok ? Abraços Pacini Em 20 de outubro de 2014 17:41, Mariana Groff < bigolingro

Re: [obm-l] Problema Legal

2014-10-20 Por tôpico Pedro José
Boa tarde! Esqueci-me do caso com 5 números. Mas o Ralph já complementou. Saudações, PJMS Em 20 de outubro de 2014 17:04, Ralph Teixeira escreveu: > E a ideia do Pedro também resolve o caso de 5 números distintos a com suas somas, que na ordem têm de ser: > (s1=a+b) < (s2=a+c) < (s3=?+?) <= (

Re: [obm-l] Problema Legal

2014-10-20 Por tôpico Ralph Teixeira
E a ideia do Pedro também resolve o caso de 5 números distintos a: > Boa tarde! > > Não havia me apercebido, mas por sorte não muda nada. > > Pois, como os números são distintos, se ordenarmo-los, a > b > c >d e as > somas s1 > s2 > s3 >= s4 > s5 > s6. > > Como os números são distintos a + b = s1,

Re: [obm-l] Problema Legal

2014-10-20 Por tôpico Pedro José
Boa tarde! Não havia me apercebido, mas por sorte não muda nada. Pois, como os números são distintos, se ordenarmo-los, a > b > c >d e as somas s1 > s2 > s3 >= s4 > s5 > s6. Como os números são distintos a + b = s1, a + c = s2, b+d = s5 e c + d = s6. logo poderemos formar um sistema: Ax = b ond

Re: [obm-l] Problema Legal

2014-10-20 Por tôpico Ralph Teixeira
Hmmm... mas cuidado: o problema não parece informar que somas correspondem a que combinações das variáveis, então tem um pouco mais do que um sistema de equações aí. Então o problema agora é o seguinte: seja s=(s1, s2, s3, ..., s6) o vetor de somas do lado direito do seu sistema. Você consegue mos

Re: [obm-l] Problema Legal

2014-10-20 Por tôpico Pedro José
Bom dia! Saiu errado a terceira linha é | 0 0 1 -1 | e não | 0 0 0 -1| conforme escrito anteriormente. Saudações, PJMS. Em 20 de outubro de 2014 09:16, Pedro José escreveu: > Boa tarde! > > (a) Ax=b| 1 1 00 | |a|| r| > |10100 | |b| |s| >

Re: [obm-l] Problema Legal

2014-10-20 Por tôpico Pedro José
Boa tarde! (a) Ax=b| 1 1 00 | |a|| r| |10100 | |b| |s| | 1 1 00 | |c| = |t | | 1 1 00 | |a| | r | Trabalhando a matriz A sem alterar seu posto, 2a = 1a - 2a; 3a = -3a + 1a -2a; 4a = 4a - 2a + 2 . 3a terem

Re: [obm-l] Problema

2014-10-19 Por tôpico Pacini Bores
Ok Maurício, obrigado. Já vi a elegante solução do Ralph. Abraços Pacini Em 19 de outubro de 2014 15:03, Mauricio Barbosa escreveu: > Oi pacini, > Acredito que seja indiferente os lados opostos a se considerar, mas não > será válida a propriedade ao mesmo tempo para os dois pares de lados >

Re: [obm-l] Problema

2014-10-19 Por tôpico Mauricio Barbosa
Oi pacini, Acredito que seja indiferente os lados opostos a se considerar, mas não será válida a propriedade ao mesmo tempo para os dois pares de lados opostos no mesmo quadrilátero, ou seja, se a+c+x =16 então não necessariamente b+d+y=16 , ou a+c+y=16. Somente um deles. Abç!!! Em 18/10/2014 17:11

Re: [obm-l] Problema

2014-10-18 Por tôpico Ralph Teixeira
Sejam a e c os dois lados mencionados, e d a diagonal. Note que a área A do quadrilátero satisfaz (pense dois triângulos, um com lados a e d, outro com lados c e d): A <= da/2 + dc/2 = d(16-d)/2 Mas esta última expressão é no máximo 32, que só ocorre quando d=8. Então tem que valer a igualdade 2A

Re: [obm-l] Problema

2014-10-18 Por tôpico Pacini Bores
Oi Maurício, sem querer enviei sem completar. Continuando : A propriedade que vc enunciou está valendo para todos os lados ? Por exemplo : a+c + x = 16 e também vale a+c+y=16 : ou a+c+x =16 e b+d+ y =16 Onde a e c são lados opostos. Abraços Pacini Em 16 de outubro de 2014 12:4

Re: [obm-l] Problema

2014-10-18 Por tôpico Pacini Bores
Oi Maurício, me tira uma dúvida no enunciado : Sejam os lados do quadrilátero a, b,c e d; e diagonais x e y. A propriedade q Em 16 de outubro de 2014 12:40, Mauricio Barbosa escreveu: > Boa tarde amigos, > alguém poderia me ajudar com o problema: > Em um quadrilátero convexo de área 32cm2,

Re: [obm-l] Problema de encaixotamento de esferas:

2014-10-14 Por tôpico Rogerio Ponce
Ola' Fabio, as esferas devem ficar em uma das diagonais principais da caixa. Assim, elas sao tangentes em um ponto sobre essa diagonal, de modo que seus centros distam 7+8=15 cm entre si. Alem disso, o centro de cada esfera fica a uma certa distancia do vertice mais proximo. Essas distancias sao sq

[obm-l] Re: [obm-l] Problema da Olimpiada de Matemática de Moscou

2014-09-03 Por tôpico Ralph Teixeira
Claramente, x<=1993. Então S(x)<=1+9+9+9=28, e portanto S(S(x))<=1+9=2+8=10. Portanto, 1993-38=1955<=x<=1993, isto é, x="19ab" onde 38<="ab"<=93. Então reestimo S(x)=1+9+a+b entre 1+9+4+0 e 1+9+8+9, isto é, em [14,27], e portanto S(S(X)) entre 2+0 e 1+9, isto é, em [2,10] Portanto, x está entre 19

Re: [obm-l] Re: [obm-l] Problema de geometria!

2014-07-07 Por tôpico Douglas Oliveira de Lima
gt; FE/DF=sqrt(3) então > > Julio Saldaña > > > -- Mensaje original --- > De : obm-l@mat.puc-rio.br > Para : obm-l@mat.puc-rio.br > Fecha : Sun, 6 Jul 2014 18:02:13 -0300 > Asunto : Re: [obm-l] Problema de geometria! > >OI Douglas , > > > >Pensando n

[obm-l] Re: [obm-l] Problema de geometria!

2014-07-07 Por tôpico Julio César Saldaña
Seja P o ponto da prolongação de CE tal que PE/EC=3 => AP//FE Seja Q o ponto da prolongação de AD tal que AD/DQ=3 => QC//DF Seja BQ=x => AB=x.sqrt(3) Seja BC=y => BP=y.sqrt(3) Então BQ/BC=x/y=AB/AP. E como FE/DF=sqrt(3) então OI Douglas , Pensando neste problema, se usar a lei dos cosse

Re: [obm-l] Problema de geometria!

2014-07-06 Por tôpico Carlos Victor
OI Douglas , Pensando neste problema, se usar a lei dos cossenos nos triângulos FCE, AFD e DBE e usando o fato de que cos(90+B)= -senB ( não é muito trabalhoso); deixando na forma de quadrados não é difícil de concluir que 4 FE^2= ED^2 e que 4DF^2 = 3ED^2 ; ou seja o triângulo EFD é retângulo e q

[obm-l] Re: [obm-l] Problema de lógica

2014-03-05 Por tôpico Ralph Teixeira
Tah errado, eh 5, 10, 2, 9, 8, 4, 6, 7, 3, 1, 0. Ordem alfabetica. 2014-03-05 16:20 GMT-03:00 Jorge Paulino : > A sequência a seguir é formada por 10 números: > 5, 10, 2, 8, 9, 4, 6, , , . > Os 3 últimos números dessa sequência são, respectivamente, > A) 1, 3 e 7. B) 1, 7 e 3. C) 3,

Re: [obm-l] Problema do Cavalo

2014-03-01 Por tôpico Celso Faria de Souza
ada em: segunda-feira, 24 de fevereiro de 2014 13:12 > Para: obm-l@mat.puc-rio.br > Assunto: Re: [obm-l] Problema do Cavalo > > Uma pergunta além: você quer saber quantas casas foram atingidas ao final > do percurso, certo? No seguinte sentido: > > No primeiro passo, ele pode

Re: [obm-l] Problema do Cavalo

2014-02-25 Por tôpico Benedito Tadeu V. Freire
No primeiro passo, existem 8 possibilidades para o cavalo atingir. -- Open WebMail Project (http://openwebmail.org) -- Original Message --- From: terence thirteen To: obm-l@mat.puc-rio.br Sent: Mon, 24 Feb 2014 13:12:27 -0300 Subject: Re: [obm-l] Problema do Cavalo >

<    1   2   3   4   5   6   7   8   9   10   >